LSAT and Law School Admissions Forum

Get expert LSAT preparation and law school admissions advice from PowerScore Test Preparation.

User avatar
 Dave Killoran
PowerScore Staff
  • PowerScore Staff
  • Posts: 5852
  • Joined: Mar 25, 2011
|
#46249
Complete Question Explanation
(The complete setup for this game can be found here: lsat/viewtopic.php?t=14384)

The correct answer choice is (E)

Similar to question #22, this question frequently forces you to account for the variables remaining for team 2.

The question stem establishes that G is assigned to team 1, and from the first rule we know that G must be in the front position. But, by itself, this information does not serve to immediately impact any of the answer choices. Thus, we must look more closely at each answer.

Answer choice (A): Because G must be assigned to the front position per the first rule, H cannot be assigned to the same team, and thus this answer choice is incorrect.

Answer choice (B): When G, K, and L are assigned to team 1, then team 2 must be composed of three members from the group of H, N, P, and Q. But, from the contrapositive of the sixth rule, since Q is not on team 1, H cannot be on team 2, leaving only N, P, and Q available for team 2. But, the fifth rule prohibits P and Q from being on the same team (or, alternately, from the fourth rule N and P cannot be on the same team), and so there are not enough jugglers available to form a viable group for team 2, and this answer choice cannot occur.

Answer choice (C): When G, K, and P are assigned to team 1, then team 2 must be composed of three members from the group of H, L, N, and Q. But, from the contrapositive of the sixth rule, since Q is not on team 1, H cannot be on team 2, and team 2 must then be composed of L, N, and Q. However, from the third rule, if L is assigned, L must be assigned to team 1, and so there are not enough jugglers available to form a viable group for team 2. Thus, this answer choice is incorrect.

Answer choice (D): When G, L, and N are assigned to team 1, then team 2 must be composed of three members from the group of H, K, P, and Q. But, from the contrapositive of the sixth rule, since Q is not on team 1, H cannot be on team 2, and team 2 must then be composed of K, P, and Q. But, the fifth rule prohibits P and Q from being on the same team, and so again there are not enough jugglers available to form a viable group for team 2, and this answer choice cannot occur.

Thus, answer choice (E) is the credited response.

Get the most out of your LSAT Prep Plus subscription.

Analyze and track your performance with our Testing and Analytics Package.